Zona de peligro para aeronaves

Declaración de la pregunta: -

Un jet enemigo vuela a una altura constante de 250  metro con una velocidad de 500  EM . El avión de combate pasa sobre un cañón antiaéreo que puede disparar en cualquier momento y en cualquier dirección con una velocidad de 100  EM . Determine el intervalo de tiempo durante el cual el avión de combate está en peligro de ser alcanzado por las balas del arma.

Fuente:- Mecánica para JEE (Principal y Avanzado) (Vol.-1) - Er. Anurag Mishra


Mi solución:-

Como sabemos la ecuación de trayectoria de un proyectil proyectado en un ángulo θ bajo la influencia de la fuerza gravitacional está dada por

(1) y = X broncearse θ gramo X 2 2 tu 2 ( 1 + broncearse 2 θ )

dónde X y y representan las coordenadas cartesianas.

Ahora, considere t = 0  segundo como el momento en que el arma antiaérea dispara una bala para golpear el avión a reacción. Entonces, para impactar en el avión, la bala debe tener las mismas coordenadas que la del avión a reacción en el momento en que la bala alcanza la misma altura que la del avión a reacción, es decir 250  metro . Entonces, supongamos que después t segundos la bala alcanza una altura de 250  metro . En ese momento las coordenadas del avión a reacción serían ( 500 t , 250 ) , entonces el X la coordenada de la bala será X = 500 t .

Entonces, al sustituir tu = 100  EM ; gramo = 10  metro/ s 2 ; y = 250  metro ; X = 500 t en ecuacion ( 1 ) , obtenemos

250 = ( 500 t ) broncearse θ gramo ( 500 t ) 2 2 × 100 2 ( 1 + broncearse 2 θ ) gramo ( 500 t ) 2 2 tu 2 broncearse 2 θ ( 500 t ) broncearse θ + ( 250 + gramo ( 500 t ) 2 2 tu 2 ) = 0

Ahora, para que exista tal trayectoria D 0

( 500 t ) 2 4 gramo ( 500 t ) 2 2 tu 2 ( 250 + gramo ( 500 t ) 2 2 tu 2 ) 0 t 2 ( 2 t 2 ) 0

El intervalo para t es t [ 2 , 2 ] ,pero como t > 0 , entonces t ( 0 , 2 ] .

Por lo tanto, el avión a reacción está en peligro de ser alcanzado por el cañón antiaéreo durante un intervalo de 2 segundos


Solución del libro: -

La ecuación de la trayectoria de las balas es

(1) y = X broncearse θ gramo X 2 2 tu 2 ( 1 + broncearse 2 θ )

zona de peligro según libroPara un valor dado de X , máximo y se puede determinar a partir de

d y d ( broncearse θ ) = X gramo X 2 tu 2 ( broncearse θ ) = 0 broncearse θ = tu 2 gramo X

Al sustituir la expresión por broncearse θ en ecuacion ( 1 ) , obtenemos

y metro a X = tu 2 gramo gramo X 2 2 tu 2 [ 1 + tu 4 gramo 2 X 2 ] y metro a X = tu 2 2 gramo gramo X 2 2 tu 2

El caparazón puede golpear un área definida por

y tu 2 2 gramo gramo X 2 2 tu 2

Al sustituir valores numéricos, y = 250 metro ; tu = 100 metro / s ; gramo = 10 metro / s 2 , obtenemos

X 2 2000 250 500 2 X 500 2

El avión de combate, puede viajar 1000 2 m mientras se puede golpear. Entonces, el avión está en peligro por un período de

1000 2 500 = 2 2 segundo


Mi trato con la pregunta: -

  1. ¿Por qué mi respuesta y la respuesta de los libros difieren? ¿Qué me estoy perdiendo?
  2. ¿Por qué la solución del libro encuentra el máximo y para un particular X (que creo que es la posición del chorro en algún momento como se explica en mi solución, corríjame si me equivoco).
  3. ¿Por qué el proyectil puede golpear solo el área definida en la solución? ¿Quiso decir que solo en esa área el proyectil puede golpear el avión? Si es así, todavía tengo la duda de por qué solo esa región.
  4. Por último, ¿cuál podría haber sido la inspiración del autor para llegar a esta solución?

Una solución más elegante siempre es bienvenida.


Edición 1: - Sé que se ha hecho esta pregunta aquí , pero como el OP no proporcionó ningún trabajo, se suspendió, por lo tanto, proporcioné mi trabajo y también, esto no es una tarea, lo estoy resolviendo en mi propio.


Edición 2:-

Como señalaron todos los solucionadores, cometí un error al determinar el tiempo de referencia para el chorro, así que para corregir eso intenté lo siguiente.

Sea la posición del chorro en el plano cartesiano ( X + 500 t , 250 ) , dónde X y Δ t representa el tiempo transcurrido después de disparar la bala del arma y la bala golpeando el avión (ya que no pude pensar en nada bueno que también pudiera decir eso) t puede ser negativo si tomamos t = 0 como el tiempo de referencia, así que en su lugar definí Δ t ). Al sustituir estos valores en la ecuación de trayectoria del proyectil obtenemos,

Ecuación de la trayectoria de un proyectil proyectado en un ángulo θ bajo la influencia de la fuerza gravitacional está dada por

(1) y = X broncearse θ gramo X 2 2 tu 2 ( 1 + broncearse 2 θ )

Al sustituir ( X + 500 t , 250 ) , en ecuación ( 1 ) , obtenemos

250 = ( X + 500 t ) broncearse θ gramo ( X + 500 t ) 2 2 tu 2 ( 1 + broncearse 2 θ ) ( 500 t + X ) 2 gramo 2 tu 2 broncearse 2 θ ( 500 t + X ) broncearse θ + ( 250 + ( 500 t + X ) 2 gramo 2 tu 2 ) = 0 ( 500 t + X ) 2 2000 broncearse 2 θ ( 500 t + X ) broncearse θ + ( 5 × 10 5 + ( 500 t + X ) 2 2000 ) = 0

Ahora bien, para que exista tal trayectoria broncearse θ R , entonces D 0

( X + 500 t ) 2 4 ( ( 500 t + X ) 2 2000 ) ( 5 × 10 5 + ( 500 t + X ) 2 2000 ) 0 ( 500 t + X ) 2 ( 5 × 10 5 ( 500 t + X ) 2 ) 0

Ahora deja ( 500 t + X ) = pag , entonces la desigualdad anterior se convierte en

pag 2 ( 5 × 10 5 pag 2 ) 0 500 2 pag 500 2

Como podemos ver que pag = ( 500 t + X ) representa el X -coordenada del chorro, por lo que obtenemos que el chorro está en peligro como se representa en la desigualdad anterior para 2 2 segundos.

Dígame si cometí algún error, y parece que la solución del libro es mucho más intuitiva y breve.

Respuestas (3)

ACTUALIZADO Sammy Gerbil señaló correctamente que mi respuesta fue incorrecta, y me disculpo. Así que aquí está la respuesta "real"...

Comencemos con el siguiente diagrama:

ingrese la descripción de la imagen aquí

Es un diagrama de las posibles trayectorias de la bala de cañón, disparada en diferentes ángulos. En verde está el ángulo "crítico", el que alcanzaría una altura de 250 m. Pero como puede ver, ¡ese NO es el ángulo que golpeará el avión en el punto más lejano posible! Es posible apuntar un poco más alto y llegar un poco más lejos (gracias Sammy Gerbil por señalar esto).

La pregunta mostró una ecuación para la trayectoria con la que no todos pueden estar familiarizados. No estaba... Así que decidí convencerme a mí mismo derivándolo de la ecuación paramétrica que conozco. Para un proyectil disparado con velocidad v en un angulo θ a la horizontal, las componentes horizontal y vertical de la velocidad son:

v X = v porque θ v y = v pecado θ gramo t

Podemos integrar estos wrt time para obtener la posición:

X = v porque θ t y = v pecado θ t 1 2 gramo t 2

Reordenando la expresión para X Nos da t :

t = X v porque θ

Sustituyendo esto en la expresión de y obtenemos

y = v pecado θ X v porque θ 1 2 gramo X 2 v 2 porque 2 θ = X broncearse θ gramo X 2 2 v 2 ( 1 + broncearse 2 θ )

Esto te resulta familiar, era nuevo para mí. Pero fue el siguiente bit donde estabas teniendo problemas. Calculó el rango de veces en que una bala podría alcanzar la altura h , y supuso que el mayor tiempo empleado correspondería al disparo más lejano posible (lo que en realidad no es cierto: el tiempo empleado sería mayor si la bala se disparara hacia arriba). También supuso que la bala se cruzaría con el avión que pasaba por encima cuando se disparó. Dado que el jet vuela más rápido que la bala, esto nunca puede ser cierto. El hecho de que su solución incluso se haya acercado al valor numérico correcto, entonces, es algo así como un milagro. [ pero mira a continuación... Creo que después de tu última edición entiendo este "milagro" ]

Así que sigamos con el razonamiento. Necesitamos encontrar la distancia más lejana posible. X que se puede alcanzar en altura h . La única variable es θ . Esto significa que si tomamos la derivada de X con respecto a θ , el punto estacionario (pendiente cero) corresponde a la distancia más lejana (hay que comprobar que no es la distancia más cercana, por supuesto). Esto es lo que hizo la respuesta en el libro. Pero (y esto es lo inteligente), reconocer que broncearse θ es una función monótona sobre el rango de valores de interés < π / 2 , π / 2 ] > , uno puede igualmente decidir tomar una derivada con respecto a broncearse θ . Por lo general, esto se haría formalmente mediante la sustitución de variables; la solución del libro toma un atajo.

si ponemos q = broncearse θ , podemos reescribir la ecuación de la trayectoria como

(1) y = X q gramo X 2 2 v 2 ( 1 + q 2 )

y el valor de q donde está el punto estacionario (alcance más lejano) ocurre cuando

d y d q = 0

Nota: con este enfoque, estamos permitiendo y variar con θ - encontramos el mayor valor posible de y en un dado X . Esto es matemáticamente más fácil de hacer porque la expresión es lineal en y . Sin embargo, esto nos lleva a una expresión para la línea roja discontinua en mi diagrama como si hubiera tomado la derivada de X con respecto a θ - pero habría sido un trabajo más duro.

Esto significa

0 = X q gramo X 2 v 2 X = q gramo X 2 v 2 q = v 2 gramo X

Sustituyendo esta expresión por q de vuelta a la trayectoria (1), obtenemos

y = v 2 gramo gramo X 2 2 v 2 ( 1 + ( v 2 gramo X ) 2 ) = v 2 2 gramo gramo X 2 2 v 2

Esto describe una parábola que es la envolvente de todos los puntos posibles que se pueden alcanzar (porque en cada valor de y nos da el más grande y el más pequeño X que se puede golpear). Agregué esa curva como una línea discontinua roja en la figura. Y ahora la solución es sencilla.

Solo tenemos que encontrar el rango de valores X que están dentro de la línea discontinua roja - en otras palabras, resolvemos

h = v 2 2 gramo gramo X 2 2 v 2 X 2 = 2 v 2 gramo ( v 2 2 gramo h )

Esto nos da dos valores para X - uno positivo y uno negativo. si configuras gramo = 10   metro / s , el intervalo es [ 500 2 , 500 2 ] y dado que el avión vuela a una velocidad de 500 m/s, es vulnerable por un total de 8 segundos: el tiempo que tarda en volar a través de la zona donde el cañón podría alcanzarlo.

Tenga en cuenta que el cañón tendrá que dispararse en todo momento considerablemente antes de que el avión entre en el "espacio aéreo protegido"; incluso si se dispara hacia arriba, tardará más de 2,5 segundos en alcanzar la altura del avión.

Para su referencia, aquí está el código de Python utilizado para generar la trama:

import numpy as np
import matplotlib.pyplot as plt
from math import pi,sqrt,acos,asin

vgun = 100.0
h    = 250.0
g    = 9.81

t = np.linspace(0,20,200)

# the curve that just touches the path of the jet:
vy_critical = sqrt(2*g*h)
th_critical = acos(vy_critical / vgun)
ax=plt.figure()
vx = vgun*np.sin(th_critical)
vy = vgun*np.cos(th_critical)
x = t*vx
y = vy*t-0.5*g*t*t
plt.plot(x,y,'g')
x = -x;
plt.plot(x,y,'g')

# add a number of trajectories at different angles:
for theta in np.linspace(-90,90,19)*pi/180:
    vx = vgun*np.sin(theta)
    vy = vgun*np.cos(theta)
    x = t*vx
    y = vy*t-0.5*g*t*t
    plt.plot(x,y,'b')


# limiting angle - largest range
th_crit = asin(sqrt((-2*g*h+vgun*vgun)/(-2*g*h+2*vgun*vgun)))
vxc = vgun*np.sin(th_crit)
vyc = vgun*np.cos(th_crit)
xc = t*vxc
yc = t*vyc -0.5*g*t*t
plt.plot(xc,yc,'m')
plt.plot(-xc,yc,'m')

# add the path of the jet:
plt.plot([-800,800],[h,h],'r')

# limit the range of the axes:
ax=plt.gca()
ax.set_ylim((0,2*h))
ax.set_xlim((-800,800))
plt.xlabel('horizontal position (m)')
plt.ylabel('vertical position (m)')
plt.title('possible trajectories')

# add the envelope of trajectories
x = np.linspace(-800,800,1000)
y = vgun*vgun/(2*g) - g*x*x/(2*vgun*vgun)
plt.plot(x,y,'r',ls='--') # red dashed line
plt.show()

ACTUALIZAR

Entonces, ¿qué es lo que está mal, si es que hay algo, con la última versión de su solución (que, después de todo, está obteniendo la respuesta correcta)? Tuve que pensarlo durante bastante tiempo, pero creo que descubrí lo que está pasando. Mi confusión no fue ayudada por el hecho de que usaste X para significar dos cosas diferentes: tanto la posición inicial del chorro cuando se dispara el cañón como la posición donde es interceptado por el cañón.

Pero es por eso que su solución funciona. Está resolviendo todas las trayectorias posibles que se cruzan y para las cuales broncearse θ es válida. Esto conduce a una desigualdad en su término pag 2 , y eso te da un valor mínimo y máximo de pag . Finalmente, desde pag = 500 t + X 0 , y tienes los dos pag metro a X y pag metro i norte , puedes encontrar la diferencia en el tiempo sin necesidad de saber qué X 0 era.

Circuitous, pero correcto.

El enfoque habitual es encontrar el valor máximo de X , tomando la derivada. Eso es lo que hicieron tanto el libro como mi solución. Pero, de hecho, encontró otra forma válida de obtener la solución. Muy bien hecho.

Entonces mi solución es correcta, el único problema es que no consideré el intervalo de tiempo negativo para disparar el cañón, y también adelgacé (corríjame si me equivoco) lo que sugiere es que no debería haber tomado t = 0 como referencia para disparar el cañón, debería haber considerado t como el intervalo de tiempo en que la bala llega al 250 metro altura y luego igualar la X coordenada de la bala a la de la posición del chorro después del paso del tiempo t .
Sí, creo que es correcto. Estuviste muy cerca, solo quería visualizar la situación de una manera que dejara más claro que puedes "disparar hacia atrás".
Ah ... una cosa más, las preguntas que he resuelto hasta ahora sugieren que el tiempo negativo sugiere que los fenómenos requeridos también habrían ocurrido en la misma trayectoria cuando el proyectil habría sido disparado antes. X segundos, donde X es el valor absoluto del tiempo negativo. Entonces, en esta pregunta, ¿cómo sabes que un tiempo negativo sugiere disparar hacia atrás en lugar de la trayectoria imaginaria? Además, si es posible, ¿puede explicar qué dice la solución del libro, porque me ha estado comiendo por dentro durante mucho tiempo?
El problema es que estás decidiendo arbitrariamente que hay un momento al que llamas "t=0". No hay necesidad de eso. En su lugar, debe ver "durante qué segmento de su vuelo el avión puede ser alcanzado por el cañón", y eso es lo que muestra mi diagrama. Luego dices "la distancia es X, la velocidad v, entonces el tiempo de peligro es t = X/v". No hay necesidad de invocar "tiempo negativo" a menos que diga "por encima del cañón es t = 0", pero eso es completamente arbitrario. Si el avión cruza ese punto a las 10 en punto, también podría decir "t = 36000".
De acuerdo... entonces uno no debe invocar un tiempo de referencia en este tipo de condiciones donde la posición de un objeto es desconocida (para el solucionador), en su lugar, lo que se necesita es el uso de un intervalo de tiempo. Corrígeme si estoy equivocado.
De acuerdo, después de pensar con calma, creo que su solución es realmente completa y responde a todas mis dudas, por lo que la aceptaré.
Esa fue una excelente respuesta, Floris. Por eso lo voté.
Creo que tu respuesta puede ser incorrecta. Si entiendo correctamente, está diciendo que el intervalo entre los dos puntos donde la línea roja y las trayectorias verdes se cruzan es la "zona de peligro". Sin embargo, las trayectorias azules inmediatamente por encima de las verdes cruzan la línea roja (por segunda vez) a una distancia horizontal mayor del cañón antiaéreo. Entonces, la "zona de peligro" me parece más grande de lo que dices.
@sammygerbil tienes razón!! Tendré que pensar esto un poco más y actualizar mi respuesta.
@sammygerbil - Lo he arreglado ahora.
@Floris: bien, ahora, después de ver su respuesta modificada, estoy confundido nuevamente. En el párrafo donde comenzaste a explicar la solución del libro, explicaste que para una altura fija (digamos h ) el maximo X para esa altura se puede obtener por d X d θ = 0 , así que para h = 250 metro , el maximo X puede ser encontrado por d X d θ = 0 , entonces ¿por qué mencionaste "y el valor de q donde está el punto estacionario (alcance más lejano) ocurre cuando d y d z = 0 ", como y es constante por lo que la derivada será 0 con seguridad. Esto fue lo que más me molestó en la solución del libro también.
No entendí cómo concluyes lo que dijiste sobre mi solución, lo que hice fue (lo cual explicaste, por qué estaba mal al igual que todos los demás solucionadores) solo la intersección de las trayectorias del avión y el arma AA, que dependía del parámetro t (tiempo), que fue muy mal elegido pero no entendí cómo concluyeste "Calculaste el rango de tiempos en los que una bala podría alcanzar la altura h , y supuso que el mayor tiempo necesario correspondería al disparo más lejano posible (lo que en realidad no es cierto: el tiempo necesario sería mayor si la bala se disparara directamente hacia arriba)".
@Floris: actualicé mi solución si puede verificar si hay algún error, también sigo confundido acerca de la solución del libro y mis dudas aún permanecen como se indica en los dos comentarios anteriores
Voy a echar otro vistazo y me pondré en contacto con usted!
¡Mira si mis últimas ediciones te aclaran las cosas!

La trayectoria del proyectil es una parábola, cuyo vértice (punto de máxima altura) depende de θ . Para valores particulares de θ , el vértice ni siquiera alcanza la altitud a la que vuela el avión enemigo. Entonces, para todos aquellos casos en los que el ápice es igual o mayor que la altitud del avión enemigo, hay un posibilidad de golpear el jet enemigo (el artillero tendrá que cronometrar su disparo correctamente). Esta es la zona de peligro para el jet enemigo, comenzando y terminando en esos valores de X , donde el vértice de la parábola es igual a la altura del chorro.

En otras palabras, siempre que la altura del vértice de la parábola y metro a X y j mi t (altitud del jet) el jet está en zona de peligro. Eso es lo que dice la solución del libro de texto, lo cual es correcto.

Veo que lo que insinúas es que siempre que y metro a X y j mi t , entonces el avión está en peligro de ser alcanzado, pero ¿qué pasa si para cuando la bala sube (o cae desde el vértice) a la altitud a la que vuela el avión, el avión ya ha escapado de su alcance (de la bala)? a la altura de 25 metro )
En la pregunta, y siguiendo también la respuesta, la zona de peligro parece definirse como esa región donde existe la mera posibilidad de ser golpeado. Si fueras piloto de ese jet, ¿no lo definirías de la misma manera?
@ user350331, ¿está seguro de que no hay un error tipográfico en la pregunta original? Como se indicó, la bala viaja 5 veces más lento que el avión. Obviamente, esto es bastante poco realista si realmente quieres derribar el avión.

En su solución, considere lo que sucede cuando t = 0 .
El proyectil está en posición ( 0 , 0 ) y el jet está en posición ( 0 , 250 ) es decir, a una altura de 250 m directamente sobre el proyectil.
Si esta fuera la condición inicial, el proyectil nunca golpearía el avión.


He cambiado la anotación del gráfico que espero les ayude a comprender cómo se obtuvo la solución.

Esa solución encuentra la altura máxima. y máximo para un desplazamiento horizontal dado del proyectil X .

y máximo = tu 2 2 gramo gramo X 2 2 tu 2 = 500 X 2 2000

ingrese la descripción de la imagen aquí

Todo lo que hay que hacer ahora es asegurarse de que la altura máxima del proyectil y máximo es igual o superior a la altura del chorro que es de 250 m.
Los valores límite de X ocurrir cuando 250 = 500 X 2 2000 X = ± 500 2 m y luego sigue la respuesta.

De su solución, lo que pude deducir fue que, en primer lugar, la zona de peligro es el área en la que probablemente se golpeará el avión a reacción si el disparo se sincroniza perfectamente. También en su solución dijo que la zona de peligro se decide de tal manera que el vértice de la trayectoria de la bala 250 metro . Entonces, si averiguamos el alcance máximo del proyectil que resulta ser 1000   metro en θ = π / 4 y en este ángulo el vértice sale a estar en 250 metro , entonces π / 4 θ π / 2 , tengo razón.
También tendría que considerar el disparo del arma en ambos cuadrantes. Y si el arma puede disparar en una inclinación π / 4 θ π / 2 entonces tengo problemas para encontrar las coordenadas cuando la bala está a una altitud 250 metro para que pueda encontrar el intervalo de X en el que el jet está en peligro. No sé por qué, pero todavía me siento un poco incómodo con esta pregunta y no soy capaz de pensar en palabras que expongan mis dudas con claridad.
@ user350331 He reescrito mi respuesta para tratar de explicar con un poco más de detalle cómo se obtuvo la respuesta del libro.
Muchas gracias.... la visualización con las anotaciones añadidas ayudó mucho y también respondieron las preguntas que quería hacer, pero no pude encontrar palabras lo suficientemente buenas para que tuvieran sentido. Tendría que confesar que eres un salvavidas.